As a , b , c range over all positive reals less than 1 satisfying 3 a b c = a 1 + b 1 + c 1 1 , find the maximum value of a 2 + 1 − b c 1 + b 2 + 1 − c a 1 + c 2 + 1 − a b 1 .
Details and assumptions
- This problem is not original.
This section requires Javascript.
You are seeing this because something didn't load right. We suggest you, (a) try
refreshing the page, (b) enabling javascript if it is disabled on your browser and,
finally, (c)
loading the
non-javascript version of this page
. We're sorry about the hassle.
There are many elegant ways to solve this, but I am not at all in the moooooooooooood .
Equality case gives the extreme values!
a = b = c gives each of the fractions to be 1 , giving answer 3 .
Your second statement is not necessarily true.
Log in to reply
Agreed, this is not there in many of the Cauchy Schwarz or other identity related question, but this one is symmetric in abc, so that was enough for bashing !
Log in to reply
First, what you did isn't called bashing, it's called guessing. And just because an expression is symmetric in its terms doesn't necessarily mean it's optimized when the terms are all equal. And even it were true, how would simply plugging the values show whether the optimum value you get is the maximum or the minimum?
Log in to reply
@Sreejato Bhattacharya – Why don't bunnies grow up :/
@sreehari vp @Sreejato Bhattacharya Can you post a solution and then help sort this into the corresponding Inequalities skill? Thanks!
Log in to reply
Done. Thanks for reminding me, sir. :)
I'm not sure to which skill set this problem should go--this is more about algebraic manipulations than applying inequalities. Could you please help decide?
I actually worked on this and showed that the expression was less than or equal to 2 + c y c l i c ∑ a 2 + 1 − b c b c . Then I guessed that that the answer was 3 . Seeing this then I tried working on a full solution by trying to show that c y c l i c ∑ a 2 + 1 − b c b c ≤ 1 . I tried that for sometime but failed. Can anyone help me from here?
@Sreejato Bhattacharya Can you help me?
Actually, the equality case given is the GM-HM inequality, in which we know that the equality case occurs when a = b = c .
Problem Loading...
Note Loading...
Set Loading...
The following solution isn't mine.
The condition is equivalent to a b + b c + c a = 3 1 .
I claim that the expression never exceeds 3 . This is equivalent to showing that ( a b + b c + c a ) ( a 2 + 1 − b c 1 + b 2 + 1 − c a 1 + c 2 + 1 − a b 1 ) ≤ 1 ⟺ cyc ∑ a 2 − b c + 1 2 ( a b + b c + c a ) ≤ 2 ⟺ 3 − cyc ∑ a 2 − b c + 1 2 a b + 2 b c + 2 c a ≥ 1 ⟺ cyc ∑ 1 − a 2 − b c + 1 2 ( a b + b c + c a ) ≥ 1 ⟺ cyc ∑ a 3 − a b c + a a 2 ( a + b + c ) ≥ 1 . By Titu's lemma, cyc ∑ a 3 − a b c + a a 2 ≥ a 3 + b 3 + c 3 − 3 a b c + a + b + c ( a + b + c ) 2 . Hence, it suffices to show that ( a + b + c ) 3 ≥ a 3 + b 3 + c 3 − 3 a b c + a + b + c , which is, in fact, an identity. Hence, we're done.
Equality holds for a = b = c = 3 1 .
Hence, our desired answer is 3 .
This problem is from a recent China TST.